LSAT and Law School Admissions Forum

Get expert LSAT preparation and law school admissions advice from PowerScore Test Preparation.

 Administrator
PowerScore Staff
  • PowerScore Staff
  • Posts: 8916
  • Joined: Feb 02, 2011
|
#40667
Complete Question Explanation
(The complete setup for this game can be found here: lsat/viewtopic.php?t=15480)

The correct answer choice is (C)

Answer choice (A): K must be shown third or fourth, and J must be shown sixth or seventh. Thus, K can never be shown after J.

Answer choice (B): L must be shown fourth or fifth, and J must be shown sixth or seventh. Thus, L can never be shown after J.

Answer choice (C): Because J could be shown sixth and P could be shown seventh, this is the correct answer choice.

Answer choice (D): If both N and O are shown after M, then there will not be enough houses to show in the morning, and hence this answer choice cannot occur.

Answer choice (E): If both N and P are shown after K, then there will not be enough houses to show in the morning, and hence this answer choice cannot occur.

Get the most out of your LSAT Prep Plus subscription.

Analyze and track your performance with our Testing and Analytics Package.